LSAT and Law School Admissions Forum

Get expert LSAT preparation and law school admissions advice from PowerScore Test Preparation.

User avatar
 Dave Killoran
PowerScore Staff
  • PowerScore Staff
  • Posts: 5852
  • Joined: Mar 25, 2011
|
#41278
Complete Question Explanation
(The complete setup for this game can be found here: lsat/viewtopic.php?t=11660)

The correct answer choice is (C)

The last question in a game is often the most difficult, and that general rule holds true here. Let us take a moment to examine why LSAC so often makes the last question difficult. The key to understanding this phenomenon is to look at it from a psychological perspective. As you near the end of a game, your mind naturally begins to focus on quickly finishing the game at hand and preparing for the next game. At just this point, when you want to go more quickly, LSAC throws in a difficult question. This tends to have the effect of slowing you down considerably, and that usually leads to a degree of frustration. Once you become frustrated, your chances of missing the question increase. And when you go to the next game, you may still be thinking about what happened on the last question, and that can contribute to a poor start on the new game, causing further trouble. In a nutshell, do not forget about the importance of psychology on the test. You must remain positive, focused, and calm throughout each section. If you become upset or frustrated, take a moment to relax and regain your equilibrium.

The local condition in the question stem sets up the following relationship:
J96_Game_#1_#7_diagram 1.png
This sequential relationship automatically produces several Not Laws:
J96_Game_#1_#7_diagram 2.png
In addition, the interaction of the last rule and the sequence further establishes that G cannot be inspected on day 3 since it would then be impossible for Q to be inspected on day 5 (in this question if G is inspected on day 3 then R would be inspected on day 5). Also, because of the block produced by the question stem, if G cannot be inspected on day 3, then Q cannot be inspected on day 4 and R cannot be inspected on day 5. Using these inferences and the Not Laws above, we can eliminate answer choices (A), (D), and (E) from consideration. At this point, unless you see what distinguishes answer choice (B) from answer choice (C), the best strategy would probably be to try a quick hypothetical using either answer choice. As it turns out, answer choice (B) is incorrect, since if H is inspected on day 6 the following impossible scenario results:
J96_Game_#1_#7_diagram 3.png
As shown above, G is inspected on day 3, but Q is not inspected on day 5—a violation of the last rule. By process of elimination, answer choice (C) is proven correct.

Some students question whether answer choice (C) can be valid since it leads to a solution (F-J-H-G-Q-R) where Q is inspected on day 5 but G is not inspected on day 3. Remember, the conditional rule only activates if G is inspected on day 3. If Q is inspected on day 5, nothing necessarily happens (to think otherwise would be a Mistaken Reversal).
You do not have the required permissions to view the files attached to this post.
User avatar
 Stephanie Oswalt
PowerScore Staff
  • PowerScore Staff
  • Posts: 811
  • Joined: Jan 11, 2016
|
#63902
Hello,

We have received the below question from a student. An instructor will respond shortly. Thank you!
Hello,
I am struggling with the above explanation for question #7*. You do not put F in your diagram and I am unsure how you knew it is supposed to go in slot #1.

Thank you for your time,
Sofie

*admin note- this same explanation also can be found on on page 172 of the LSAT Logic Games Bible 2018 edition
 Adam Tyson
PowerScore Staff
  • PowerScore Staff
  • Posts: 5153
  • Joined: Apr 14, 2011
|
#63908
In addition to the explanation above, here's a link to the setup for this game, which will help establish some of the baseline parameters:

lsat/viewtopic.php?t=11660

For this question, we have a rather unwieldy block of GQR, which has to be after J somewhere. One approach to this question is described at the top of this thread as a process of elimination, but another way to attack is to identify the few places that big block can go. We know it cannot start at space 1, because J is before it, but let's see what happens if it starts at space 2, forcing J to space 1:

JGQR_ _

Since F must be either 1st or 6th, in this case F must be 6th, and that leaves H, the random variable, to go 5th, for this solution:

JGQRHF

With this in hand, scan the answers to see if any of them match. Remember, this is a Could Be True question, so we only need to find one thing that is possible.

Rats, no matches! Okay, where else can that block go? We cannot start it at 3, as explained earlier in this thread, because that would put Q at 4, but when G is 3rd Q must be 5th. So, let's shift it down one more space, starting the block at the 4th space:

_ _ _ GQR

This now forces F to be 1st, since 6th is occupied, and J and H become interchangeable at 2nd and 3rd, like so:

F J/H H/J GQR

Again, scan the answers, and boom! Answer C is possible! None of the others work.

So, to answer your question, we didn't play around with F in the explanation because there was a solution where it went first and another where it went last. Instead, we focused on eliminating impossible answers to select what was left. Taking this alternate approach is about searching for what can happen rather than eliminating what cannot, and either is a viable option. It's up to you to determine which one is more intuitive for you and best matches your personal style when it comes to games. Try both and see what you like!
 Coleman
  • Posts: 44
  • Joined: Jul 07, 2020
|
#83758
I understand there are two options to go with F - JGQRHF and FJHGQR.
However, I wonder why HJGQRF or FJGQRH is impossible. In these two scenarios, it is hard to narrow down the answer choice into (C) exclusively since it is "could be true" question.
 Adam Tyson
PowerScore Staff
  • PowerScore Staff
  • Posts: 5153
  • Joined: Apr 14, 2011
|
#83763
You're forgetting a rule there, Coleman! As discussed earlier in this thread, the GQR block cannot go at 345 because if G is 3rd, Q must be 5th. That rule still applies to this local question! Both of your proposed solutions violate that rule, because you have G 3rd and Q 4th.

Also, note that another possible solution is FHJGQR. The H and the J can swap places when the GQR block is placed in spots 456.
User avatar
 akreimerman1
  • Posts: 14
  • Joined: Jul 16, 2023
|
#102404
Hi,

I am looking at the explanations for this homework lesson but I do not understand why C is the correct answer, I do not understand the explanation by giving a scenario that is impossible yet the question stem asks for something that could be true? Can someone help me understand?

Thank you!
 Adam Tyson
PowerScore Staff
  • PowerScore Staff
  • Posts: 5153
  • Joined: Apr 14, 2011
|
#102407
Did you review the explanation and further discussion in this thread, akreimerman1?

viewtopic.php?f=360&t=13722

The short answer is that C is correct because all the other answers are impossible.

C works because a solution of FJHGQR is viable.

There are also two other possible solutions when G is immediately before Q, and those are:

FHJGQR (just flipping the positions of J and H)

and

JGQRHF

Was there another answer choice that you felt worked, or did C look like it presented a problem? Let us know what you thought!
User avatar
 akreimerman1
  • Posts: 14
  • Joined: Jul 16, 2023
|
#102413
Hi Adam,

I was just a little bit confused by the GQR block in this question and the global condition of G3 -> Q5 and couldn't figure out how to mix them both. Your explanation helped, thank you!

Get the most out of your LSAT Prep Plus subscription.

Analyze and track your performance with our Testing and Analytics Package.